What reason would you use to prove ΔFGH ≅ ΔJHK ?
Select one:
ASA
HL
SAS
cannot prove congruency
SAA

What Reason Would You Use To Prove FGH JHK ?Select One:ASAHLSAScannot Prove CongruencySAA

Answers

Answer 1

The reason that is used to prove ΔFGH ≅ ΔJHK is SAS property

If the three angles and the three sides of a triangle are equal to the corresponding angles and the corresponding sides of another triangle, then both the triangles are said to be congruent.

The two triangles need to be of the same size and shape to be congruent. Both the triangles under consideration should superimpose on each other.

According to the question,

Given : (1) ∠FGH and ∠JHK are right angles

(2) H is midpoint of GK

(3) GH ≅ JK

In ΔFGH and ΔJHK

∠FGH = ∠JHK ( right angles )GH ≅ JK (given) GH = KH ( H is midpoint of GK )

Therefore ,  ΔFGH  ≅ ΔJHK using  SAS property

Hence , The reason use to prove ΔFGH ≅ ΔJHK is SAS

To know more about Congruent triangles here

https://brainly.com/question/27848509

#SPJ4


Related Questions

Graph: y < -1

Is (-2,5) a solution? yes or no
Is ( 6,2) a solution? yes or no

Answers

Answer: Both points are not a solution.

Step-by-step explanation:

        This says that the graph will be every y-value less than -1. Both 5 (in the point (-2, 5)) and 2 (in the point (6, 2)) are greater than -1, meaning they are both not a solution.

        See attached for a graph with the coordinate points graphed as well. Since these points are not in the shaded region, they are not solutions.

Harry and Sally both invest $2500 for their
school tuition in 3 years. Harry chooses to
invest his money at bank A at 2.5%/a
compounded monthly while Sally chooses
to invest her money at bank B at 2.75%/a
compounded quarterly. How much money
will Harry and Sally have in their bank
account after 3 years? Who made the best
investment? Explain your answer.

Answers

The amount Harry has in his account is $2,694.50

The amount Sally has in his account is $2,714.23.

Sally made the best investment.

What is compound interest?

It is the interest we earned on the interest.

The formula for the amount earned with compound interest after n years is given as:

A = P [tex](1 + r/n)^{nt}[/tex]

P = principal

R = rate

t = time in years

n = number of times compounded in a year.

We have,

Harry:

P = $2500

T = 3 years

r = 2.5%

n = 12

A = 2500 [tex](1 + 0.208)^{36}[/tex]

A = 2500 x [tex]1.208^{36}[/tex]

A = $2,694.50

Sally:

P = $2500

T = 3 years

r = 2.75%

n = 4

A = 2500 [tex](1 + 0.69)^{12}[/tex]

A = 2500 x [tex]1.69^{12}[/tex]

A = $2,714.23

Thus,

Harry has $2,694.50 in his account.

Sally has $2,714.23 in his account.

Sally made the best investment.

Learn more about compound interest here:

https://brainly.com/question/13155407

#SPJ1

Solve:
19) m∠2 = 12x-4

Answers

For the given isosceles triangle when we solve m < 2 = 12x - 4 we get the value of x = 4.5 degrees.

It is known that two sides are equal so it is a isosceles triangle so the two angles are equal, that is

m<2 = 50

50 = 12x - 4

50 + 4 = 12x

54 = 12x

Divide by 12 on both sides

54 / 12 = 12 x / 12

x = 54 / 12

= 27 / 6

= 9 / 2

= 4.5 degrees

Learn more about isosceles triangle here

https://brainly.com/question/2456591

#SPJ4

To find a value that is in an unordered array of 100 items, how many values must linear search examine on average?
a) 101
b) 100
c) 7
d) 50
e) 10

Answers

Answer: D its 50

Step-by-step explanation:

i need help asap!!! please help me i don’t know how to do this!!

Answers

The answer is 45
Hope this helps

PLSS can you help me and answers

Answers

Answer #23

The lines will intersect at (1, 8) therefore this system of equations has one solution. This can be found by graphing or solving by elimination method. I attached a graph for you.



Answer # 24

False, the lines have the same slope but different y intercepts so the lines are parallel, they will never intersect and they have no solution

whats the geometric mean of 6 and 28

Answers

Answer:

Geometric mean: 12.961481396816

TRUE/FALSE. when we conduct time series forecasting it is safest to utilize regression analysis, because then we will not be extrapolating.

Answers

When we conduct time series forecasting it is safest to utilize regression analysis, because then we will not be extrapolating the above statement is False.

Regression analysis is a group of analytical techniques used to measure the relationships between a dependent variable and one or more independent variables. It may be used to simulate the long-term link between variables and gauge how strongly the relationships between them are related.

Models that evaluate the connection between a dependent variable and an independent variable include simple linear regression. The following equation represents the simple linear model:

Y = a + bX + ϵ

Regression analysis is the most secure method for time series forecasting since it involves extrapolation.

For more such question on regression analysis.

https://brainly.com/question/7781679

#SPJ4

Triangles U V W and X Y Z are shown. Angles V U W and Z X Y are congruent, angles U W V and X Z P are congruent, and angles U V W and Z Y X are congruent. The length of side U V is 50, the length of V W is 60, and the length of U W is 40. The length of side Z Y is 48, the length of Y X is 40, and the length of Z X is 32.
Which statement best describes triangles UVW and XYZ?

They are similar and congruent.
They are similar, but not congruent.
They are congruent, but not similar.
They are neither congruent nor similar.

Answers

The statement that best describes triangles UVW and XYZ is; They are similar, but not congruent.

How to Identify Congruent Angles?

We are given that;

Angles VUW and ZXY are congruent.

Angles UWV and XZP are congruent.

Angles UVW and ZYX are congruent.

Now, the length of side UV is 50, the length of VW is 60, and the length of UW is 40. The length of side ZY is 48, the length of YX is 40, and the length of ZX is 32.

Here the triangles are similar by the AAA similarity and the length of the first triangle is in the ratio 40:50:60 = 4:5:6 .

The side of the second triangle is also in the ratio 32:40:48 = 4:5:6 .

Therefore, the triangles UVW and XYZ are similar but not congruent.

Read more about Congruent Angles at; https://brainly.com/question/1675117

#SPJ1

Answer:

B. They are similar, but not congruent

Find Mean, Median, and Mode
What is the mean, median, and mode of the data in this set?
0.1,12.3,2.4,10.1,4.5,6.2

Answers

For the data in the given set , the mean is 7.6  , the median is 8.15  and the median is 10.1   .

In the question ,

the data in the set is given as 10.1 , 12.3 , 2.4 , 10.1 , 4.5 , 6.2  .

on arranging data set in ascending order ,

we get , 2.4 , 4.5 , 6.2 , 10.1 , 10.1 , 12.3  .

the sum is = 45.6

the mean is = sum /(number of terms)

= 45.6/6

= 7.6  

the data has 6 numbers of terms

So ,the median will be (3rd term + 4th term)/2

= (6.2 + 10.1)/2

= 16.3/2

= 8.15

the term 10.1 occurs most number of time that is two times .

So , the mode is 10.1 .

Therefore , the mean is 7.6 , median is 8.15 and mode is 10.1   .

Learn more about Mean here

https://brainly.com/question/3183994

#SPJ4

Name: Sally Jonas, single, 1 exemption.
Pay Rate: $5.95 per hour
Hours Worked: M-8, T-7, W-8, T-9, F-8

Total Hours Worked :
Wage for Week :
FICA Tax (7.65 %) :
Federal Tax :
State Income Tax (at 3%) :
City Income Tax (1.5%) :
Total Tax :
Paycheck after Taxes :

Answers

She worked 40 hours total and earned $238

Total hours worked:           $40

Wage for Week:                 $238

FICA Tax (7.65%):               $18.21

Federal Tax (from Table):    $41.60

State Income Tax (at 3%):    $7.14

City Income Tax (1.5%):        $3.57

Total Tax:                            $70.52

Paycheck after Taxes:         $167.48

Describe the transformation of the parent
function f(x) = x³, to obtain the function
g(x) = (x − 8)³ + 4.

Answers

The transformation from the parent function is a translation by 8 units right and 4 units up

How to describe the transformation from the parent function?

From the question, we have the following function that can be used in our computation:

f(x) = x³

g(x) = (x − 8)³ + 4

First, we have the transformation to be:

From f(x) = x³ to f'(x) = (x − 8)³

This means the function is translated right by 8 units

Next, we have:

From f'(x) = (x − 8)³ to g(x) = (x − 8)³ + 4

This means the function is translated up by 4 units

Hence, the transformation is 8 units right and 4 units up

Read more about transformation at

brainly.com/question/27224272

#SPJ1

et h be the set of all points of the form (s, s-1). determine whether h is a vector space. if it is not a vector space, determine which of the following properties it fails to satisfy. a: contains zero vector b: closed under vector addition c: closed under multiplication by scalars

Answers

No, h is not a vector space. It fails to satisfy property (b), closed under vector addition.

For a set to be a vector space, it must satisfy three properties: contains the zero vector, closed under vector addition and closed under multiplication by scalars.

For a set to be closed under vector addition, the sum of any two vectors in the set must also be in the set. In this case, the set h contains the points (s, s-1).

When two points are added, the result is (2s, 2s-2), which is not in the set h. Therefore, h fails to satisfy property (b) and is not a vector space.

Learn more about vector here

https://brainly.com/question/15709504

#SPJ4

the updated forecast for the amazon sales accounting for trend, seasonality and adjusted for the serial correlation using the ar(1) model is: billion dollars. if the answer is 75.2432 billion dollars just type 75.243 keep at least 3 decimal places for all parts of your calculation. record your answer with at least 3 decimal places.

Answers

As per the forecast, the updated forecast for the amazon sales accounting for trend, seasonality and adjusted for the serial correlation using the AR(1) model is 75.243.

In statistics, the term correlation defines the degree to which two variables move in coordination with one another.

Here we have Quarterly Amazon Sales (billion dollars) starting in Quarter 3 of 2013 through Quarter 2 of 2021 . And assume in Quarter 2 of 2021 the Amazon Sales were 96.6 billion dollars.

Now we have to use the output provided below to report the updated forecast for the Amazon Sales for Quarter 3 of 2021.

Here the variable time period is coded in the following way:

Start with

t=1 for Quarter 3 of 2013,

t=2 for Quarter 4 of 2013 .

Then the value of t will increase by 1 for each subsequent quarter.

Here in this example we have created indicator variables for Quarters 1, 2 and 3. Quarter 4 is the base level.

Now, by using the output here for the model accounting for trend and seasonality as well as the output for the AR(1) model report the updated forecast for Amazon Sales in Quarter 3 of 2021.

Here the subset of the output posted below: Response Revenues Bivariate Fit of Residual Revenues $2 By Lag residuals Parameter Estimates

Therefore, the Updated forecast for the Amazon Sales accounting for Trend, Seasonality and Adjusted for the Serial Correlation using the AR(1) model is 75.243

To know more about Correlation here.

https://brainly.com/question/28898177

#SPJ4

How do I find the area between the curves of y=x+2 and y=x^{2}-2?

Answers

Refer to the attached photos. I would appreciate a rate :)

What is the midpoint overline AB with A (2, 4) and B (- 6, 8) Mid Point Formula ((x_{1} + x_{2})/2, (y_{1} + y_{2})/2)

Answers

If the midpoint of AB with A (2, 4) and B (- 6, 8) is at point  (-2, 6)

How to find the midpoint

The formula to determine the center point of a straight line using the coordinates of its endpoints is known as the midpoint formula in coordinate geometry.

The formula is of the form

X= (x₂ + x₁)/2

Y = (y₂ + y₁)/2

solving for the midpoint

X= (2 - 6) / 2

X = -4 / 2

X = -2

Y = (y₂ + y₁)/2

Y = (4 + 8) / 2

Y = 12 / 2

Y = 6

The midpoint is (-2, 6)

Learn more about midpoint  here:

https://brainly.com/question/29730021

#SPJ1

Evaluate the expression using De Moivre’s Theorem: √2+3i

Give your answer in a+bi form.

Answers

Answer:

cos(π/2)+isin(π/2)

Step-by-step explanation:

hopefully that's the answer you're looking for, haven't done problems like this in a while-

consider joining our discørd server for additional free hw help! we also have an active giveaway rn. code: RPEdT3erFm :)

which ones have no solution?

Answers

Answer:b,d

Step-by-step explanation:

Solve For X, using the degrees given in the equation.

Answers

Answer:

x= 101°.

Step-by-step explanation:

1. Refer to the theory.

Theory of triangles states that the sum of all inner angles of a triangle should equal 180°. Hence, you can make an equation that gives the missing value of "x" in this problem.

2. Form the equation.

[tex]x+30+49=180[/tex]

3. Solve the equation.

[tex]x=180-30-49\\ \\x=101[/tex]

4. Conclude.

x= 101°.

A football match consists of two 45 minute halves and a 30 minute break in between. How many minutes does the whole match last for?

Answers

The whole match lasts for 45 minutes + 45 minutes + 30 minutes = 120 minutes. Answer: 120 min.

At the West Texas Balloon Festival in El Paso, a hot air balloon started its descent from an altitude of 800 feet. After 1 minute, it is at a height of 700 feet and continues at this rate till it reaches the ground.

A. Write a linear equation to find the height, h of the balloon after m minutes.

B. What is the rate of change. What does it represent in this situation?

C. What is the y-intercept and what does it represent?

D. What is the x-intercept and what does it represent?

Answers

The linear equation and the definition of the parts of the equation  of the motion of the balloon are as follows;

A. The equation is; h = 800 - 100·m

B. The rate of change is 100 ft/min. The rate of change represents the height the balloon descends in one minute

C. The y-intercept is 800 (feet). The y-intercept represents the initial height of the balloon

D. The x-intercept is 8 (minutes). The x-intercept represents the time it takes the balloon to reach the ground

What is a linear equation?

A linear equation is one that has a straight line graph.

The initial height of the balloon = 800 feet

Height of the balloon after 1 minute = 700 feet

A. The change in height of the balloon in one minute = 100 feet

The rate of change of the height of the balloon = -100 feet per minute (the height reduces by 100 feet after each minute)

The linear equation for the balloon, can therefore be obtained from the following details;

The initial height of the balloon = The y-intercept of the linear equation, c = 800 feet

The rate of change of the height of the balloon = The slope of the linear equation, m = -100 ft/min

The linear equation in the form, y = m·x + c, that can be used to find the  height of the balloon is therefore;

h = -100·m + 800

h = 800 - 100·m

B. The rate of change is the rate at which the height of the balloon changes, which is 100 feet per minute

The rate of change represents the slope of the linear equation and the rate at which the height of the balloon reduces.

C. The y-intercept is the point at which the linear equation intersects the y-axis which is the point at which the input variable, m, is zero, indicating the height at which the balloon started its descent.

Therefore, the y-intercept represents the initial height of the balloon

D. The x-intercept is the point at which the graph intersects the x-axis, and at which the y-value (value of the output variable) is zero, and the x-intercept can be specified by the x-value at the point of the intersection.

At the x-intercept, therefore, h is zero, which is the ground level, and m is the time it takes the balloon to reach the ground.

At the x-intercept, therefore, we het;

h = 0 = 800 - 100·m

100·m = 800

m = 800 ÷ 100 = 8

The x-intercept is m = 8 minutes

The x-intercept therefore, represents the time (8 minutes) it takes the balloon to reach the ground.

Learn more about graphs of linear equations here:

https://brainly.com/question/13954699

#SPJ1

help please
One angle measures 25° and another angle measures (3d − 10)°. If the angles are complementary, what is the value of d?


d = 55

d = 35

d = 25

d = 11.7

Answers

If the angles are complementary, the value of d is C. d = 25.

What are complementary angles?

Complementary angles are two angles whose measures add up to 90° . In the figure below, ∠1 and ∠2 are complementary. Two angles need not be adjacent to be complementary. When the sum of two angles is equal to 90 degrees, they are called complementary angles. For example, 30 degrees and 60 degrees are complementary angles

In this case, it will be:

25 + 3d - 10 = 90

3d + 15 = 90

Collect like terms

3d = 90 - 15.

3d = 75.

Divide

d = 75/3

d = 25

The correct option is C.

Learn more about angles on:

https://brainly.com/question/25716982

#SPJ1

The product of 9 and the difference of a number and 7.
Answer:
I
Submit Answer

Answers

Answer:

9x-63

Step-by-step explanation:

9*(x-7)

9x-63

I need help with this question!!

Answers

Answer:

  (b)  0.058 or 5.8%

Step-by-step explanation:

Given a normal distribution with mean 10.72 and standard deviation 0.14, you want the probability that a value is less than 10.5.

Probability calculator

A suitable probability calculator needs to know the mean, standard deviation, and set of values of interest. Here, ...

  mean = 10.72

  standard deviation = 0.14

  P(X < 10.5)

The calculator shows it to be 0.058, or 5.8%.

2x5−x3+x2+4; reflection in the y-axis and a vertical stretch by a factor of 3, followed by a translation 1 unit down

Answers

3x2-3(y+1)+9x2+12; reflection in the y-axis, a vertical stretch by a factor of 3, and a translation 1 unit down

The reflection in the y-axis can be achieved by changing the sign of the x-terms while leaving the y-term unchanged. Afterwards, the function needs to be stretched vertically by a factor of 3. This means that all terms will be multiplied by 3. Lastly, the function requires a translation 1 unit down. This can be done by adding a -1 to the y-term. Therefore, the final equation is 3x2-3(y+1)+9x2+12 which includes the reflection in the y-axis, vertical stretch by a factor of 3, and translation 1 unit down.

Learn more about reflexive here

https://brainly.com/question/14492876

#SPJ4

A red and a blue die are thrown. Both dice are fair. The events A, B, and C are defined as follows: A: The sum on the two dice is even
B: The sum on the two dice is at least 10
C. The red die comes up 5

Answers

The probability of event A occurring is 1/2, the probability of event B occurring is 1/3, and the probability of event C occurring is 1/6.

in order for a sum of two dice to be even, the numbers on the two dice must add up to an even number. The possible combinations of two dice that add up to an even number are (1,3), (3,1), (2,4), (4,2), (5,5). Therefore, the probability of event A occurring is 1/2.

In order for the sum of two dice to be at least 10, the numbers on the two dice must add up to 10, 11, 12. The possible combinations of two dice that add up to 10, 11, 12 are (4,6), (6,4), (5,5), (5,6), (6,5). Therefore, the probability of event B occurring is 1/3.

In order for the red die to come up 5, the number on the other die does not matter. Therefore, the probability of event C occurring is 1/6.

Learn more about probability here

https://brainly.com/question/11234923

#SPJ4

The ultimate goal of muted group theory is to:A. Help men understand the perspective of women. B. Elect women to major leadership positions in Western democracies. C. Change linguistic systems that make it more difficult for women to be heard. D. Predict how women and men will communicate across gender lines.C. Change linguistic systems that make it more difficult for women to be heard.(The feminist dictionary is one attempt to create an alternative vocabulary.)

Answers

The question mentions muted group theory, which is a kind of Communication Theory. The answer is option C, which changes linguistic systems that make it more difficult for women to be heard.

A verbal system is a language taken as a combination of its corridor. A verbal system can also relate to a set of choices that druggies of language have in contriving rulings.

Muted group proposition is the introductory idea, articulated by anthropologists Ardener and Ardener, that in every society there are artistic groups who are traditionally muted — given lower access than members of the dominant groups to public converse and to having their individual and community enterprises heard.

Muted group proposition is the introductory idea, that in every society there are artistic groups who are traditionally muted — given lower access than members of the dominant groups to public converse and to having their individual and community enterprises heard.

To know more about Communication Theory,

brainly.com/question/27962320

brainly.com/question/28583454

#SPJ4

Round 3.122 to the nearest hundredth

Answers

Answer:

3.12

Step-by-step explanation:

1 number after the decimal - tenths place.

2 numbers after the decimal - hundredths place.

3 numbers after the decimal - thousandths place.

4 numbers after the decimal - ten thousandths place.

5 numbers after the decimal - hundred thousandths place.

Answer:

3.12

Step-by-step explanation:

3.122 rounds up to 3.12 because 2 is less than 5, so the number rounds down.

What’s the answer? pls hurry!

Answers

Answer:

Step-by-step explanation:

use (Y2-Y1)/(X2-X1)

(7-2)/(0-(-5))=5/5=1

y=1x+b is your equation with value b as the y intercept.
Plug in a point into this and solve for b

7=1(0)+b
7=b

final equation is y=x+7

A is the answer


(20x³+12x² +17x-17)÷(5x²-2x)

Answers

The division of the given polynomial is [tex]\frac{20x^3+12x^2+17x-17}{5x^2-2x}=4x+4+\frac{25x-17}{5x^2-2x}[/tex].

What is a long division of polynomials?

A long-division polynomial is an algorithm for dividing polynomials by another polynomial of the same or a lower degree. The long division of polynomials also consists of the divisor, quotient, dividend, and the remainder as in the long division method of numbers.

The given expression is (20x³ + 12x² + 17x - 17) ÷ (5x² - 2x)

[tex]=\frac{20x^3+12x^2+17x-17}{5x^2-2x}\\\\=4x+\frac{20x^2+17x-17}{5x^2-2x}\\\\=4x+4+\frac{25x-17}{5x^2-2x}\\[/tex]

Hence, the division of the given polynomial is [tex]\frac{20x^3+12x^2+17x-17}{5x^2-2x}=4x+4+\frac{25x-17}{5x^2-2x}[/tex].

To learn more about the long division of polynomials, visit:

https://brainly.com/question/27601809

#SPJ1

Other Questions
You work at the headquarters of an enterprise known for unethical practices. The company has many remote sites, but most functions are performed at one location. Your enterprise recently hired a third-party vendor known for high-accuracy business impact analyses. The BIA performed by the vendor has since proved wrong, as an incident impacted the business significantly more than forecast. You are assigned to conduct a study on the BIA's misconception and submit a report. What should you investigate as the possible reason for the BIA's inaccuracy? :1) The vendor overlooked the organization's remote sites.2) The vendor was unaware of the organization's unethical practices.3) The vendor was unaware of some of the organization's business concepts.4) The vendor used the incorrect method to conduct their analysis. assume victoria's indifference curves are bowed outward but her indifference curves satisfy the other three properties of indifference curves. as victoria moves from right to left along the horizontal axis, her marginal rate of substitution suppose you observe a binary system containing a main-sequence star and a brown dwarf. the orbital period of the system is 1 year, and the average separation of the system is 1 au . you then measure the doppler shifts of the spectral lines from the main-sequence star and the brown dwarf, finding that the orbital speed of the brown dwarf in the system is 23 times greater than that of the main-sequence star.1. How massive is the brown dwarf?Mdwarf = ? KG __________ models seek to determine quantities of different products to produce and sell in order to maximize contribution to profit.a. Product mixb. Process selectionc. Linear optimizationd. Linear regression PLEASE HELP 40 POINT PLEASE 40 POINTS ASAPWomen became more active outside the home during the late 18th and early 19th centuries as a result of:A.the passage of laws granting women voting rights.B.court rulings giving women more control over property.C.the election of the first female governor.D.the growth of the temperance movement. a municipality can protect itself against the construction or repair of properties that do not conform to the building codes established by ordinance by requiring the contractors to obtain which of the following theories posits that when a person experiences something with a negative result, such as pain or discomfort, aggressive behavior can often occur in the wake of that experience? An investor purchases zero-coupon bonds issued by the U.S. Treasury due to mature in 18 years at $100,000. Which of the following might describe the primary reason for selecting that investment vehicle?1. The investor is 65 years old and needs the reliability of current income.2. The investor is 45 years old and has purchased these in an IRA rollover account and wants the assurance of funds for retirement.3. The investor is 30 years old and has a newborn child and wishes to assure funds for a college education.4. The investor is 20 years old, has just received an inheritance, and wishes to shelter income for as long as possible. what are the hypothesis and the conclusion of the conditional? if anangle measures 130, then the angle is obtuse Given m|| n, find the value of x and y.(8x+1)Z(4y+3)(9x-16)m72Use photo for better explanation PLEASE HELP 32. Write a rule to represents the function(2, 10), (4, 20), (5, 25), (7, 35), (9, 45) today most large corporations not only accept the necessity of affirmative action but also find that benefits when they make themselves more diverse? Hope one of yall help me with this question.. because i was struggling!!Please and thank you. the total mechanical energy of a system as a function of time is shown in the graph. which of the following statements is true regarding the system? -28 = -4xWhat value of x makes this equation true?A -7B 24C-32D 7 in remote desktop connection, which tab controls which resources in the local computer are available for the session? a 23-year-old man was struck across the face with a baseball bat. his eyes are swollen shut, he has massive facial bruising and deformities, and he has blood in his mouth. your most immediate concern should be: Which of these is a potential job for Lucia? calculate the molar solubility of srf2 (ksp = 4.3109) in the following substances. Which of the following lipid molecules possess a different fundamental structural make-up from the others? A. Fatty acids B. Cholesterol C. Triglycerides D. SphingolipidsE. Glycolipids